Math, asked by rathodmeghna93, 3 months ago

G.P 1, 1/a, 1/a2, 1/a3 ___ then nth term is__​

Answers

Answered by MaheswariS
3

\textbf{Given:}

\textsf{G.P is}

\mathsf{1,\dfrac{1}{a},\dfrac{1}{a^2},\dfrac{1}{a^3},\;.\;.\;.\;.}

\textbf{To find:}

\textsf{n th term of the G.P}

\textbf{Solution:}

\textsf{Consider,}

\mathsf{1,\dfrac{1}{a},\dfrac{1}{a^2},\dfrac{1}{a^3},\;.\;.\;.\;.}

\mathsf{Here,}

\mathsf{a=1,\;r=\dfrac{1}{a}}

\textsf{The n th term of the G.P is}

\boxed{\mathsf{t_n=a\,r^{n-1}}}

\mathsf{t_n=(1)\left(\dfrac{1}{a}\right)^{n-1}}

\implies\boxed{\mathsf{t_n=\dfrac{1}{a^{n-1}}}}

\textbf{Find more:}

Find the four numbers forming a G.P if the second number exceedsthe first by 21 and the fourth number is 336 more than the third.

https://brainly.in/question/34915708

Similar questions